2009 AMC 12B Problems/Problem 14

Revision as of 18:09, 1 March 2009 by Misof (talk | contribs) (New page: == Problem == Five unit squares are arranged in the coordinate plane as shown, with the lower left corner at the origin. The slanted line, extending from <math>(a,0)</math> to <math>(3,3)<...)
(diff) ← Older revision | Latest revision (diff) | Newer revision → (diff)

Problem

Five unit squares are arranged in the coordinate plane as shown, with the lower left corner at the origin. The slanted line, extending from $(a,0)$ to $(3,3)$, divides the entire region into two regions of equal area. What is $a$? [asy] unitsize(1cm); defaultpen(linewidth(.8pt)+fontsize(8pt));  fill((2/3,0)--(3,3)--(3,1)--(2,1)--(2,0)--cycle,gray);  xaxis("$x$",-0.5,4,EndArrow(HookHead,4)); yaxis("$y$",-0.5,4,EndArrow(4));  draw((0,1)--(3,1)--(3,3)--(2,3)--(2,0)); draw((1,0)--(1,2)--(3,2)); draw((2/3,0)--(3,3));  label("$(a,0)$",(2/3,0),S); label("$(3,3)$",(3,3),NE); [/asy]$\textbf{(A)}\ \frac12\qquad \textbf{(B)}\ \frac35\qquad \textbf{(C)}\ \frac23\qquad \textbf{(D)}\ \frac34\qquad \textbf{(E)}\ \frac45$

Solution

For $a\geq 1.5$ the shaded area is at most $1.5$, which is too little. Hence $a<1.5$, and therefore the point $(2,1)$ is indeed inside the shaded part, as shown in the picture.

Then the area of the shaded part is one less than the area of the triangle with vertices $(a,0)$, $(3,0)$, and $(3,3)$. Its area is obviously $\frac{3(3-a)}2$, therefore the area of the shaded part is $\frac{7-3a}{2}$.

The entire figure has area $5$, hence we want the shaded part to have area $\frac 52$. Solving for $a$, we get $a=\boxed{\frac 23}$.

See Also

2009 AMC 12B (ProblemsAnswer KeyResources)
Preceded by
Problem 13
Followed by
Problem 15
1 2 3 4 5 6 7 8 9 10 11 12 13 14 15 16 17 18 19 20 21 22 23 24 25
All AMC 12 Problems and Solutions